2.6 HW 6

  2.6.1 Section 40, Problem 1
  2.6.2 Section 40, Problem 3
  2.6.3 Section 41, Problem 3
  2.6.4 Section 42, Problem 4
  2.6.5 Section 42, Problem 5
  2.6.6 Section 42, Problem 8
  2.6.7 Section 43, Problem 1
  2.6.8 Section 44, Problem 2
  2.6.9 Section 49, Problem 2
PDF (letter size)
PDF (legal size)

2.6.1 Section 40, Problem 1

pict
Figure 2.65:Problem statement

Solution

The PDE to solve is \[ u_{tt}=ku_{xx}\] With boundary conditions\begin{align} u\left ( 0,t\right ) & =0\tag{1}\\ Ku_{x}\left ( \pi ,t\right ) & =A\nonumber \end{align}

And initial conditions\[ u\left ( x,0\right ) =0 \] The solution to example 2 section 40 is\begin{equation} U\left ( x,t\right ) =\sum _{n=1}^{\infty }B_{2n-1}\exp \left ( \frac{-\left ( 2n-1\right ) ^{2}k}{4}t\right ) \sin \left ( \frac{\left ( 2n-1\right ) x}{2}\right ) \tag{2} \end{equation} With\[ B_{2n-1}=\frac{2}{\pi }\int _{0}^{\pi }f\left ( x\right ) \sin \left ( \frac{\left ( 2n-1\right ) x}{2}\right ) dx \] Now, in this problem, we start by writing \begin{equation} u\left ( x,t\right ) =U\left ( x,t\right ) +\Phi \left ( x\right ) \tag{3} \end{equation} The function \(\Phi \left ( x\right ) \) needs to satisfy the nonhomogeneous B.C. (1). Let \[ \Phi \left ( x\right ) =c_{1}x+c_{2}\] When \(x=0\) this gives \(0=c_{2}\). Hence \(\Phi \left ( x\right ) =c_{1}x\). Taking derivative gives \(\Phi ^{\prime }\left ( x\right ) =c_{1}\). But from (1) \(K\Phi ^{\prime }\left ( \pi \right ) =A\). Hence \(c_{1}=\frac{A}{K}\). Therefore\[ \Phi \left ( x\right ) =\frac{A}{K}x \] Substituting the above back into (3) gives\[ u\left ( x,t\right ) =U\left ( x,t\right ) +\frac{A}{K}x \] But \(U\left ( x,t\right ) \) is given by (2), hence the above becomes\begin{equation} u\left ( x,t\right ) =\frac{A}{K}x+\sum _{n=1}^{\infty }B_{2n-1}\exp \left ( \frac{-\left ( 2n-1\right ) ^{2}k}{4}t\right ) \sin \left ( \frac{\left ( 2n-1\right ) x}{2}\right ) \tag{4} \end{equation} At \(t=0\), the initial conditions is \(0\). Hence the above becomes\[ -\frac{A}{K}x=\sum _{n=1}^{\infty }B_{2n-1}\sin \left ( \frac{\left ( 2n-1\right ) x}{2}\right ) \] Hence\(\ B_{2n-1}\) is the Fourier sine series of\(\ -\frac{A}{K}x\) given by\begin{align*} B_{2n-1} & =\frac{2}{\pi }\int _{0}^{\pi }\left ( -\frac{A}{K}x\right ) \sin \left ( \frac{\left ( 2n-1\right ) x}{2}\right ) dx\\ & =-\frac{2A}{\pi K}\int _{0}^{\pi }x\sin \left ( \frac{\left ( 2n-1\right ) x}{2}\right ) dx \end{align*}

Integration by parts. Let \(u=x,dv=\sin \left ( \frac{\left ( 2n-1\right ) x}{2}\right ) \), hence \(du=1\) and \(v=-\frac{2}{\left ( 2n-1\right ) }\cos \left ( \frac{\left ( 2n-1\right ) x}{2}\right ) \) and the above becomes\begin{align*} B_{2n-1} & =-\frac{2A}{\pi K}\left ( \left [ -\frac{2x}{\left ( 2n-1\right ) }\cos \left ( \frac{\left ( 2n-1\right ) x}{2}\right ) \right ] _{0}^{\pi }+\int _{0}^{\pi }\frac{2}{\left ( 2n-1\right ) }\cos \left ( \frac{\left ( 2n-1\right ) x}{2}\right ) dx\right ) \\ & =-\frac{2A}{\pi K}\left ( -\frac{2}{\left ( 2n-1\right ) }\left [ x\cos \left ( \frac{\left ( 2n-1\right ) x}{2}\right ) \right ] _{0}^{\pi }+\frac{4}{\left ( 2n-1\right ) ^{2}}\left [ \sin \left ( \frac{\left ( 2n-1\right ) x}{2}\right ) \right ] _{0}^{\pi }\right ) \\ & =-\frac{2A}{\pi K}\left ( -\frac{2\pi }{\left ( 2n-1\right ) }\cos \left ( \frac{\left ( 2n-1\right ) \pi }{2}\right ) +\frac{4}{\left ( 2n-1\right ) ^{2}}\sin \left ( \frac{\left ( 2n-1\right ) \pi }{2}\right ) \right ) \end{align*}

Since \(2n-1\) is odd, then the cosine terms above vanish and the above simplifies to\begin{align*} B_{2n-1} & =-\frac{A}{\pi K}\frac{8\left ( -1\right ) ^{n+1}}{\left ( 2n-1\right ) ^{2}}\\ & =\frac{A}{\pi K}\frac{8\left ( -1\right ) ^{n+2}}{\left ( 2n-1\right ) ^{2}}\\ & =\frac{A}{\pi K}\frac{8\left ( -1\right ) ^{n}}{\left ( 2n-1\right ) ^{2}} \end{align*}

Substituting the above in (4) gives\begin{align*} u\left ( x,t\right ) & =\frac{A}{K}x+\sum _{n=1}^{\infty }\frac{A}{\pi K}\frac{8\left ( -1\right ) ^{n}}{\left ( 2n-1\right ) ^{2}}\exp \left ( \frac{-\left ( 2n-1\right ) ^{2}k}{4}t\right ) \sin \left ( \frac{\left ( 2n-1\right ) x}{2}\right ) \\ & =\frac{A}{K}\left \{ x+\frac{8}{\pi }\sum _{n=1}^{\infty }\frac{\left ( -1\right ) ^{n}}{\left ( 2n-1\right ) ^{2}}\exp \left ( \frac{-\left ( 2n-1\right ) ^{2}k}{4}t\right ) \sin \left ( \frac{\left ( 2n-1\right ) x}{2}\right ) \right \} \end{align*}

Which is the result required.

2.6.2 Section 40, Problem 3

pict
Figure 2.66:Problem statement

Solution

The PDE is \[ v_{t}=kv_{xx}-bv \] With boundary conditions\begin{align*} v_{x}\left ( 0,t\right ) & =0\\ v_{x}\left ( c,t\right ) & =0 \end{align*}

And initial conditions\[ v\left ( x,0\right ) =f\left ( x\right ) \] Let \(v\left ( x,t\right ) =X\left ( x\right ) T\left ( t\right ) \). Substituting into the PDE gives\[ T^{\prime }X=kX^{\prime \prime }T-bXT \] Dividing by \(XT\neq 0\) gives\begin{align*} \frac{T^{\prime }}{T} & =k\frac{X^{\prime \prime }}{X}-b\\ \frac{T^{\prime }}{T}+b & =k\frac{X^{\prime \prime }}{X}\\ \frac{T^{\prime }}{kT}+\frac{b}{k} & =\frac{X^{\prime \prime }}{X}=-\lambda \end{align*}

Where \(\lambda \) is the separation constant. We obtain the boundary value eigenvalue ODE as\begin{align} X^{\prime \prime }+\lambda X & =0\tag{1}\\ X^{\prime }\left ( 0\right ) & =0\nonumber \\ X^{\prime }\left ( c\right ) & =0\nonumber \end{align}

And the time ODE as\begin{align*} \frac{T^{\prime }}{kT}+\frac{b}{k} & =-\lambda \\ T^{\prime }+\frac{b}{k}kT & =-\lambda kT\\ T^{\prime }+\frac{b}{k}kT+\lambda kT & =0\\ T^{\prime }+T\left ( b+\lambda k\right ) & =0 \end{align*}

Now we solve the space ODE (1) in order to determine the eigenvalues \(\lambda \).

Case \(\lambda <0\)

The solution to (1) becomes\begin{align*} X\left ( x\right ) & =A\cosh \left ( \sqrt{-\lambda }x\right ) +B\sinh \left ( \sqrt{-\lambda }x\right ) \\ X^{\prime } & =A\sqrt{-\lambda }\sinh \left ( \sqrt{-\lambda }x\right ) +B\sqrt{-\lambda }\cosh \left ( \sqrt{-\lambda }x\right ) \end{align*}

Satisfying \(X^{\prime }\left ( 0\right ) =0\) gives\[ 0=B\sqrt{-\lambda }\] Hence \(B=0\) and the solution becomes \(X\left ( x\right ) =A\cosh \left ( \sqrt{-\lambda }x\right ) \). Therefore \(X^{\prime }=A\sqrt{-\lambda }\sinh \left ( \sqrt{-\lambda }x\right ) \). Satisfying \(X^{\prime }\left ( c\right ) =0\) gives\[ 0=A\sqrt{\lambda }\sinh \left ( \sqrt{-\lambda }c\right ) \] But \(\sinh \) is zero only when its argument is zero, which is not the case here since \(\lambda \neq 0\). This implies \(A=0\), leading to trivial solution. Therefore \(\lambda <0\) is not possible.

Case \(\lambda =0\)

The solution to (1) becomes\begin{align*} X\left ( x\right ) & =Ax+B\\ X^{\prime } & =A \end{align*}

Satisfying \(X^{\prime }\left ( 0\right ) =0\) gives\[ 0=A \] And the solution becomes \(X\left ( x\right ) =B\). Therefore \(X^{\prime }=0\). Satisfying \(X^{\prime }\left ( c\right ) =0\) gives\[ 0=0 \] Which is valid for any \(B\). Hence choosing \(B=1\) shows that \(\lambda =0\) is valid eigenvalue with corresponding eigenfunction \(X_{0}\left ( x\right ) =1\).

Case \(\lambda >0\)

The solution to (1) becomes\begin{align*} X\left ( x\right ) & =A\cos \left ( \sqrt{\lambda }x\right ) +B\sin \left ( \sqrt{\lambda }x\right ) \\ X^{\prime } & =-A\sqrt{\lambda }\sin \left ( \sqrt{\lambda }x\right ) +B\sqrt{\lambda }\cos \left ( \sqrt{\lambda }x\right ) \end{align*}

Satisfying \(X^{\prime }\left ( 0\right ) =0\) gives\[ 0=B\sqrt{\lambda }\] Hence \(B=0\) and the solution becomes \(X\left ( x\right ) =A\cos \left ( \sqrt{\lambda }x\right ) \). Therefore \(X^{\prime }=-A\sqrt{\lambda }\sin \left ( \sqrt{\lambda }x\right ) \). Satisfying \(X^{\prime }\left ( c\right ) =0\) gives\[ 0=-A\sqrt{\lambda }\sin \left ( \sqrt{\lambda }c\right ) \] For nontrivial solution we want\begin{align} \sin \left ( \sqrt{\lambda }c\right ) & =0\nonumber \\ \sqrt{\lambda }c & =n\pi \qquad n=1,2,3,\cdots \nonumber \\ \lambda _{n} & =\left ( \frac{n\pi }{c}\right ) ^{2} \tag{2} \end{align}

And the corresponding eigenfunctions\begin{equation} X_{n}\left ( x\right ) =\cos \left ( \sqrt{\lambda _{n}}x\right ) \tag{3} \end{equation} Now that we found \(\lambda _{n}\), we can solve the time ODE \(T^{\prime }+T\left ( b+\lambda k\right ) =0\). The solution is\begin{equation} T_{n}\left ( t\right ) =e^{-\left ( b+\lambda _{n}k\right ) t} \tag{4} \end{equation} Hence the fundamental solution is \begin{align*} v_{n}\left ( x,t\right ) & =X_{n}\left ( x\right ) T_{n}\left ( t\right ) \\ & =\cos \left ( \sqrt{\lambda _{n}}x\right ) e^{-\left ( b+\lambda _{n}k\right ) t} \end{align*}

And the general solution is the superposition of all these solutions\begin{align*} v\left ( x,t\right ) & =A_{0}X_{0}T_{0}+\sum _{n=1}^{\infty }A_{n}X_{n}\left ( x\right ) T_{n}\left ( t\right ) \\ & =A_{0}e^{-bt}+\sum _{n=1}^{\infty }A_{n}\cos \left ( \sqrt{\lambda _{n}}x\right ) e^{-\left ( b+\lambda _{n}k\right ) t} \end{align*}

Which can be written as\[ v\left ( x,t\right ) =u\left ( x,t\right ) e^{-bt}\] Where \(u\left ( x,t\right ) \) is \[ u\left ( x,t\right ) =A_{0}+\sum _{n=1}^{\infty }A_{n}\cos \left ( \sqrt{\lambda _{n}}x\right ) e^{-\lambda _{n}kt}\] Which is the same as given in section 36, page 106. In the above \begin{align*} \lambda _{0} & =0\\ \lambda _{n} & =\left ( \frac{n\pi }{c}\right ) ^{2}\qquad n=1,2,3,\cdots \end{align*}

2.6.3 Section 41, Problem 3

   Part (a)
   Part (b)

pict

pict
Figure 2.67:Problem statement

Solution

The heat PDE in spherical coordinates, assuming no dependency on \(\phi \) nor on \(\theta \) is given by\begin{align} u_{t} & =k\nabla ^{2}u\tag{1}\\ & =k\frac{1}{r}\left ( ru\right ) _{rr}\nonumber \end{align}

Where \(1<r<2\) and \(t>0\). With the boundary conditions\begin{align*} u\left ( 1,t\right ) & =0\\ u\left ( 2,0\right ) & =u_{0} \end{align*}

And initial conditions\[ u\left ( r,0\right ) =0 \]

Part (a)

Let \(v\left ( r,t\right ) =ru\left ( r,t\right ) \). Hence \(v_{t}=ru_{t}\) and \(\frac{1}{r}\left ( ru\right ) _{rr}=\frac{1}{r}v_{rr}\). Substituting these in(1), the PDE simplifies to\begin{equation} v_{t}=kv_{rr} \tag{2} \end{equation} And the boundary conditions \(u\left ( 1,t\right ) =0\) becomes \(v\left ( 1,t\right ) =0\) and \(u\left ( 2,0\right ) =u_{0}\) becomes \(v\left ( 2,t\right ) =2u_{0}\). And initial conditions \(u\left ( r,0\right ) =0\) becomes \(v\left ( r,0\right ) =0\). Hence the new boundary conditions\begin{align*} v\left ( 1,t\right ) & =0\\ v\left ( 2,t\right ) & =2u_{0} \end{align*}

And new initial conditions\[ v\left ( r,0\right ) =0 \] Now let \(s=r-1\). Since \(\frac{\partial r}{\partial s}=1\), then the PDE becomes \(v_{t}=kv_{ss}\). When \(r=1\), then \(s=0\) and the boundary conditions \(v\left ( 1,t\right ) =0\) becomes \(v\left ( 0,t\right ) =0\) and the boundary conditions \(v\left ( 2,t\right ) =2u_{0}\) becomes \(v\left ( 1,t\right ) =2u_{0}\). And initial conditions do not change. Hence the new problem is to solve for \(v\left ( s,t\right ) \) in\begin{align} v_{t} & =kv_{ss}\tag{3}\\ v\left ( 1,t\right ) & =0\nonumber \\ v\left ( 1,t\right ) & =2u_{0}\nonumber \\ v\left ( s,0\right ) & =0\nonumber \end{align}

With \(0<s<1\) and \(t>0\).

Part (b)

The PDE (3) in part(a) is now the same as result of problem 2 section 40. Hence we can use that solution for (3) which gives\[ v\left ( s,t\right ) =2u_{0}\left [ x+\frac{2}{\pi }\sum _{n=1}^{\infty }\frac{\left ( -1\right ) ^{n}}{n}e^{-n^{2}\pi ^{2}kt}\sin \left ( n\pi s\right ) \right ] \] Replacing \(s\) by \(r-1\) in the above gives\[ v\left ( r,t\right ) =2u_{0}\left [ \left ( r-1\right ) +\frac{2}{\pi }\sum _{n=1}^{\infty }\frac{\left ( -1\right ) ^{n}}{n}e^{-n^{2}\pi ^{2}kt}\sin \left ( n\pi \left ( r-1\right ) \right ) \right ] \] But \(v\left ( r,t\right ) =ru\left ( r,t\right ) \), hence \(u\left ( r,t\right ) =\frac{v}{r}\) and therefore\begin{align*} u\left ( r,t\right ) & =2u_{0}\left [ \frac{\left ( r-1\right ) }{r}+\frac{2}{\pi r}\sum _{n=1}^{\infty }\frac{\left ( -1\right ) ^{n}}{n}e^{-n^{2}\pi ^{2}kt}\sin \left ( n\pi \left ( r-1\right ) \right ) \right ] \\ & =2u_{0}\left [ \left ( 1-\frac{1}{r}\right ) +\frac{2}{\pi r}\sum _{n=1}^{\infty }\frac{\left ( -1\right ) ^{n}}{n}e^{-n^{2}\pi ^{2}kt}\sin \left ( n\pi \left ( r-1\right ) \right ) \right ] \end{align*}

Which is the result required.

2.6.4 Section 42, Problem 4

pict
Figure 2.68:Problem statement

Solution

Using method of eigenfunction expansion (or method of variation of parameters as the book calls it), we start by assuming the solution to the PDE \(u_{t}=ku_{xx}+q\left ( x,t\right ) \) is given by \begin{equation} u\left ( x,t\right ) =\sum _{n=1}^{\infty }a_{n}\left ( t\right ) \Phi _{n}\left ( x\right ) \tag{1} \end{equation} Where \(\Phi _{n}\left ( x\right ) \) are the eigenfunctions associated with the homogeneous PDE \(u_{t}=ku_{xx}\) with the homogeneous boundary conditions \(u\left ( 0,t\right ) =0\) and \(u\left ( c,t\right ) =0\). But we solved this homogeneous PDE before. It has eigenvalues and corresponding eigenfunctions\begin{align*} \lambda _{n} & =\left ( \frac{n\pi }{c}\right ) ^{2}\qquad n=1,2,3,\cdots \\ \Phi _{n}\left ( x\right ) & =\sin \left ( \sqrt{\lambda _{n}}x\right ) \end{align*}

Substituting (1) into the original PDE \(u_{t}=ku_{xx}+q\left ( x,t\right ) \) results in\begin{align*} \frac{\partial }{\partial t}\sum _{n=1}^{\infty }a_{n}\left ( t\right ) \Phi _{n}\left ( x\right ) & =k\frac{\partial ^{2}}{\partial x^{2}}\sum _{n=1}^{\infty }a_{n}\left ( t\right ) \Phi _{n}\left ( x\right ) +q\left ( x,t\right ) \\ \sum _{n=1}^{\infty }a_{n}^{\prime }\left ( t\right ) \Phi _{n}\left ( x\right ) & =k\sum _{n=1}^{\infty }a_{n}\left ( t\right ) \Phi _{n}^{\prime \prime }\left ( x\right ) +q\left ( x,t\right ) \end{align*}

But from the Sturm-Liouville ODE, we know that \(\Phi _{n}^{\prime \prime }\left ( x\right ) +\lambda _{n}\Phi _{n}\left ( x\right ) =0\). Hence \(\Phi _{n}^{\prime \prime }\left ( x\right ) =-\lambda _{n}\Phi _{n}\left ( x\right ) \) and the above reduces to\begin{equation} \sum _{n=1}^{\infty }a_{n}^{\prime }\left ( t\right ) \Phi _{n}\left ( x\right ) =-k\sum _{n=1}^{\infty }a_{n}\left ( t\right ) \lambda _{n}\Phi _{n}\left ( x\right ) +q\left ( x,t\right ) \tag{2} \end{equation} Since the eigenfunctions \(\Phi _{n}\left ( x\right ) \) are complete, we can expand \(q\left ( x,t\right ) \) using them. Therefore \[ q\left ( x,t\right ) =\sum _{n=1}^{\infty }b_{n}\left ( t\right ) \Phi _{n}\left ( x\right ) \] Substituting the above back in (2) gives\[ \sum _{n=1}^{\infty }a_{n}^{\prime }\left ( t\right ) \Phi _{n}\left ( x\right ) =-k\sum _{n=1}^{\infty }a_{n}\left ( t\right ) \lambda _{n}\Phi _{n}\left ( x\right ) +\sum _{n=1}^{\infty }b_{n}\left ( t\right ) \Phi _{n}\left ( x\right ) \] Since \(\Phi _{n}\left ( x\right ) \) are never zero, we can simplify the above to\begin{align*} a_{n}^{\prime }\left ( t\right ) & =-ka_{n}\left ( t\right ) \lambda _{n}+b_{n}\left ( t\right ) \\ a_{n}^{\prime }\left ( t\right ) +ka_{n}\left ( t\right ) \lambda _{n} & =b_{n}\left ( t\right ) \end{align*}

The above is first order ODE in \(I_{n}\left ( t\right ) \). It is linear ODE. The integrating factor is \(\mu =e^{\int k\lambda _{n}dt}=e^{k\lambda _{n}t}\). Multiplying the above ODE by this integrating factor gives\[ \frac{d}{dt}\left ( a_{n}\left ( t\right ) e^{k\lambda _{n}t}\right ) =b_{n}\left ( t\right ) e^{k\lambda _{n}t}\] Integrating both sides\begin{align*} a_{n}\left ( t\right ) e^{k\lambda _{n}t} & =\int _{0}^{t}b_{n}\left ( \tau \right ) e^{k\lambda _{n}\tau }d\tau \\ a_{n}\left ( t\right ) & =\int _{0}^{t}b_{n}\left ( \tau \right ) e^{-k\lambda _{n}\left ( t-\tau \right ) }d\tau \end{align*}

Now that we found \(a_{n}\left ( t\right ) \), we substitute it back into (1) which gives\begin{equation} u\left ( x,t\right ) =\sum _{n=1}^{\infty }\left ( \int _{0}^{t}b_{n}\left ( \tau \right ) e^{-k\lambda _{n}\left ( t-\tau \right ) }d\tau \right ) \Phi _{n}\left ( x\right ) \tag{3} \end{equation} What is left is to find \(b_{n}\left ( t\right ) \). Since \(q\left ( x,t\right ) =\sum _{n=1}^{\infty }b_{n}\left ( t\right ) \Phi _{n}\left ( x\right ) \), then by orthogonality we obtain\begin{align*} \int _{0}^{c}q\left ( x,t\right ) \Phi _{m}\left ( x\right ) dx & =\int _{0}^{c}\sum _{n=1}^{\infty }b_{n}\left ( t\right ) \Phi _{n}\left ( x\right ) \Phi _{m}\left ( x\right ) dx\\ & =\sum _{n=1}^{\infty }b_{n}\left ( t\right ) \int _{0}^{c}\Phi _{n}\left ( x\right ) \Phi _{m}\left ( x\right ) dx\\ & =b_{m}\left ( t\right ) \int _{0}^{c}\Phi _{m}^{2}\left ( x\right ) dx\\ & =b_{m}\left ( t\right ) \frac{c}{2} \end{align*}

Hence\[ b_{n}\left ( t\right ) =\frac{2}{c}\int _{0}^{c}q\left ( x,t\right ) \Phi _{m}\left ( x\right ) dx \] Substituting this back into (3) gives\begin{align} u\left ( x,t\right ) & =\sum _{n=1}^{\infty }\left ( \int _{0}^{t}e^{-k\lambda _{n}\left ( t-\tau \right ) }\frac{2}{c}\left ( \int _{0}^{c}q\left ( x,\tau \right ) \Phi _{m}\left ( x\right ) dx\right ) d\tau \right ) \Phi _{n}\left ( x\right ) \nonumber \\ & =\frac{2}{c}\sum _{n=1}^{\infty }\left ( \int _{0}^{t}e^{-k\lambda _{n}\left ( t-\tau \right ) }\left ( \int _{0}^{c}q\left ( x,\tau \right ) \Phi _{m}\left ( x\right ) dx\right ) d\tau \right ) \Phi _{n}\left ( x\right ) \tag{4} \end{align}

If we let \[ I_{n}\left ( t\right ) =\int _{0}^{t}e^{-k\lambda _{n}\left ( t-\tau \right ) }\left ( \int _{0}^{c}q\left ( x,\tau \right ) \Phi _{m}\left ( x\right ) dx\right ) d\tau \] Then (4) becomes\[ u\left ( x,t\right ) =\frac{2}{c}\sum _{n=1}^{\infty }I_{n}\left ( t\right ) \Phi _{n}\left ( x\right ) \] Since \(\Phi _{n}\left ( x\right ) =\sin \left ( \frac{n\pi }{c}x\right ) \) then the above is\[ u\left ( x,t\right ) =\frac{2}{c}\sum _{n=1}^{\infty }I_{n}\left ( t\right ) \sin \left ( \frac{n\pi }{c}x\right ) \] Which is what required to show.

2.6.5 Section 42, Problem 5

pict
Figure 2.69:Problem statement

Solution

The solution in problem \(4\) above us\begin{equation} u\left ( x,t\right ) =\frac{2}{c}\sum _{n=1}^{\infty }I_{n}\left ( t\right ) \sin \left ( \frac{n\pi }{c}x\right ) \tag{1} \end{equation} Where \[ I_{n}\left ( t\right ) =\int _{0}^{t}e^{-k\lambda _{n}\left ( t-\tau \right ) }\left ( \int _{0}^{c}q\left ( x,\tau \right ) \sin \left ( \frac{n\pi }{c}x\right ) dx\right ) d\tau \] And \(\lambda _{n}=\left ( \frac{n\pi }{c}\right ) ^{2}\). Let \(c=1,k=1\) and \(q\left ( x,t\right ) =xp\left ( t\right ) \), then the above becomes\[ I_{n}\left ( t\right ) =\int _{0}^{t}e^{-n^{2}\pi ^{2}\left ( t-\tau \right ) }\left ( \int _{0}^{1}xp\left ( \tau \right ) \sin \left ( n\pi x\right ) dx\right ) d\tau \] Substituting this in (1), using \(c=1\), then (1) becomes\begin{align} u\left ( x,t\right ) & =2\sum _{n=1}^{\infty }\left ( \int _{0}^{t}e^{-n^{2}\pi ^{2}\left ( t-\tau \right ) }\left ( \int _{0}^{1}xp\left ( \tau \right ) \sin \left ( n\pi x\right ) dx\right ) d\tau \right ) \sin \left ( n\pi x\right ) \nonumber \\ & =2\sum _{n=1}^{\infty }\left ( \int _{0}^{t}p\left ( \tau \right ) e^{-n^{2}\pi ^{2}\left ( t-\tau \right ) }\left ( \int _{0}^{1}x\sin \left ( n\pi x\right ) dx\right ) d\tau \right ) \sin \left ( n\pi x\right ) \tag{2} \end{align}

But \(\int _{0}^{1}x\sin \left ( n\pi x\right ) dx\) can now be integrated by parts. Let \(u=x,dv=\sin \left ( n\pi x\right ) \), hence \(du=1,v=-\frac{\cos \left ( n\pi x\right ) }{n\pi }\) and therefore\begin{align*} \int _{0}^{1}x\sin \left ( n\pi x\right ) dx & =-\frac{1}{n\pi }\left [ x\cos \left ( n\pi x\right ) \right ] _{0}^{1}+\frac{1}{n\pi }\int _{0}^{1}\cos \left ( n\pi x\right ) dx\\ & =-\frac{1}{n\pi }\cos \left ( n\pi \right ) +\frac{1}{n\pi }\left [ \frac{\sin \left ( n\pi x\right ) }{n\pi }\right ] _{0}^{1}\\ & =-\frac{1}{n\pi }\left ( -1\right ) ^{n}+\frac{1}{n^{2}\pi ^{2}}\left [ \sin \left ( n\pi \right ) \right ] \\ & =\frac{\left ( -1\right ) ^{n+1}}{n\pi } \end{align*}

Substituting this back in (2) gives\begin{align*} u\left ( x,t\right ) & =2\sum _{n=1}^{\infty }\left ( \int _{0}^{t}p\left ( \tau \right ) e^{-n^{2}\pi ^{2}\left ( t-\tau \right ) }\left ( \frac{\left ( -1\right ) ^{n+1}}{n\pi }\right ) d\tau \right ) \sin \left ( n\pi x\right ) \\ & =\frac{2}{\pi }\sum _{n=1}^{\infty }\frac{\left ( -1\right ) ^{n+1}}{n}\sin \left ( n\pi x\right ) \left ( \int _{0}^{t}p\left ( \tau \right ) e^{-n^{2}\pi ^{2}\left ( t-\tau \right ) }d\tau \right ) \end{align*}

Which is the solution for problem 1.

2.6.6 Section 42, Problem 8

pict

pict
Figure 2.70:Problem statement

Solution

The PDE to solve is \[ u_{t}=ku_{xx}+ax^{2}\] With boundary conditions\begin{align*} u_{x}\left ( 0,t\right ) & =0\\ u_{x}\left ( c,t\right ) & =0 \end{align*}

And initial conditions\[ u\left ( x,0\right ) =0 \] Using method of eigenfunction expansion, we start by assuming the solution to the PDE \(u_{t}=ku_{xx}+ax^{2}\) is given by \begin{equation} u\left ( x,t\right ) =\sum _{n=0}^{\infty }a_{n}\left ( t\right ) \Phi _{n}\left ( x\right ) \tag{1} \end{equation} Where \(\Phi _{n}\left ( x\right ) \) are the eigenfunctions associated with the homogeneous PDE \(u_{t}=ku_{xx}\) with the homogeneous boundary conditions \(u_{x}\left ( 0,t\right ) =0\) and \(u_{x}\left ( c,t\right ) =0\). But we solved this homogeneous PDE before. It has eigenvalues and corresponding eigenfunctions\begin{align*} \lambda _{0} & =0\\ \Phi _{0}\left ( x\right ) & =1\\ \lambda _{n} & =\frac{n^{2}\pi ^{2}}{c^{2}}\qquad n=1,2,3,\cdots \\ \Phi _{n}\left ( x\right ) & =\cos \left ( \frac{n\pi }{c}x\right ) \end{align*}

Substituting (1) into the original PDE \(u_{t}=ku_{xx}+ax^{2}\) results in\begin{align*} \frac{\partial }{\partial t}\sum _{n=0}^{\infty }a_{n}\left ( t\right ) \Phi _{n}\left ( x\right ) & =k\frac{\partial ^{2}}{\partial x^{2}}\sum _{n=0}^{\infty }a_{n}\left ( t\right ) \Phi _{n}\left ( x\right ) +ax^{2}\\ \sum _{n=0}^{\infty }a_{n}^{\prime }\left ( t\right ) \Phi _{n}\left ( x\right ) & =k\sum _{n=0}^{\infty }a_{n}\left ( t\right ) \Phi _{n}^{\prime \prime }\left ( x\right ) +ax^{2} \end{align*}

But from the Sturm-Liouville ODE, we know that \(\Phi _{n}^{\prime \prime }\left ( x\right ) +\lambda _{n}\Phi _{n}\left ( x\right ) =0\). Hence \(\Phi _{n}^{\prime \prime }\left ( x\right ) =-\lambda _{n}\Phi _{n}\left ( x\right ) \) and the above reduces to\begin{equation} \sum _{n=0}^{\infty }a_{n}^{\prime }\left ( t\right ) \Phi _{n}\left ( x\right ) =-k\sum _{n=0}^{\infty }a_{n}\left ( t\right ) \lambda _{n}\Phi _{n}\left ( x\right ) +ax^{2} \tag{2} \end{equation} Since the eigenfunctions \(\Phi _{n}\left ( x\right ) \) are complete, we can expand \(ax^{2}\) using them. Therefore \[ ax^{2}=\sum _{n=0}^{\infty }b_{n}\left ( x\right ) \Phi _{n}\left ( x\right ) \] Substituting the above back in (2) gives\[ \sum _{n=0}^{\infty }a_{n}^{\prime }\left ( t\right ) \Phi _{n}\left ( x\right ) =-k\sum _{n=0}^{\infty }a_{n}\left ( t\right ) \lambda _{n}\Phi _{n}\left ( x\right ) +\sum _{n=0}^{\infty }b_{n}\left ( x\right ) \Phi _{n}\left ( x\right ) \] Since \(\Phi _{n}\left ( x\right ) \) are never zero, we can simplify the above to\begin{align*} a_{n}^{\prime }\left ( t\right ) & =-ka_{n}\left ( t\right ) \lambda _{n}+b_{n}\left ( x\right ) \\ a_{n}^{\prime }\left ( t\right ) +ka_{n}\left ( t\right ) \lambda _{n} & =b_{n}\left ( x\right ) \end{align*}

The above is first order ODE in \(I_{n}\left ( t\right ) \). It is linear ODE. The integrating factor is \(\mu =e^{\int k\lambda _{n}dt}=e^{k\lambda _{n}t}\). Multiplying the above ODE by this integrating factor gives\[ \frac{d}{dt}\left ( a_{n}\left ( t\right ) e^{k\lambda _{n}t}\right ) =b_{n}\left ( x\right ) e^{k\lambda _{n}t}\] Integrating both sides\begin{align} a_{n}\left ( t\right ) e^{k\lambda _{n}t} & =b_{n}\left ( x\right ) \int _{0}^{t}e^{k\lambda _{n}\tau }d\tau \nonumber \\ a_{n}\left ( t\right ) & =b_{n}\left ( x\right ) \int _{0}^{t}e^{-k\lambda _{n}\left ( t-\tau \right ) }d\tau \tag{3} \end{align}

What is left is to find \(b_{n}\left ( x\right ) \). Since \(ax^{2}=\sum _{n=0}^{\infty }b_{n}\left ( x\right ) \Phi _{n}\left ( x\right ) \), and from example 1 section 8, we found that\begin{align*} b_{0}\left ( x\right ) & =a\frac{c^{2}}{3}\\ b_{n}\left ( x\right ) & =a\frac{4c^{2}}{\pi ^{2}}\frac{\left ( -1\right ) ^{n}}{n^{2}}\qquad n=1,2,3,\cdots \end{align*}

Hence when \(n=0\), then (3) becomes (since \(\lambda _{0}=0\))\begin{align*} a_{0}\left ( t\right ) & =a\frac{c^{2}}{3}\int _{0}^{t}d\tau \\ & =\frac{ac^{2}}{3}t \end{align*}

When \(n>0\) then (3) becomes\begin{align*} a_{n}\left ( t\right ) & =\left ( a\frac{4c^{2}}{\pi ^{2}}\frac{\left ( -1\right ) ^{n}}{n^{2}}\right ) \int _{0}^{t}e^{-k\lambda _{n}\left ( t-\tau \right ) }d\tau \\ & =\frac{\left ( -1\right ) ^{n}}{n^{2}}\frac{4ac^{2}}{\pi ^{2}}\int _{0}^{t}e^{-k\left ( \frac{n\pi }{c}\right ) ^{2}\left ( t-\tau \right ) }d\tau \\ & =\frac{\left ( -1\right ) ^{n}}{n^{2}}\frac{4ac^{2}}{\pi ^{2}}e^{-k\left ( \frac{n\pi }{c}\right ) ^{2}t}\int _{0}^{t}e^{k\left ( \frac{n\pi }{c}\right ) ^{2}\tau }d\tau \\ & =\frac{\left ( -1\right ) ^{n}}{n^{2}}\frac{4ac^{2}}{\pi ^{2}}e^{-k\left ( \frac{n\pi }{c}\right ) ^{2}t}\left [ \frac{e^{k\left ( \frac{n\pi }{c}\right ) ^{2}\tau }}{k\left ( \frac{n\pi }{c}\right ) ^{2}}\right ] _{0}^{t}\\ & =\frac{\left ( -1\right ) ^{n}}{n^{2}}\frac{4ac^{2}}{\pi ^{2}}\frac{e^{-k\left ( \frac{n\pi }{c}\right ) ^{2}t}}{k\left ( \frac{n\pi }{c}\right ) ^{2}}\left [ e^{k\left ( \frac{n\pi }{c}\right ) ^{2}t}-1\right ] \\ & =\frac{\left ( -1\right ) ^{n}}{n^{2}}\frac{4ac^{2}}{\pi ^{2}}\frac{1-e^{-k\left ( \frac{n\pi }{c}\right ) ^{2}t}}{k\frac{n^{2}\pi ^{2}}{c^{2}}}\\ & =\frac{\left ( -1\right ) ^{n}}{n^{4}}\frac{4ac^{4}}{k\pi ^{4}}\left ( 1-e^{-k\left ( \frac{n\pi }{c}\right ) ^{2}t}\right ) \end{align*}

Now that we found \(a_{n}\left ( t\right ) \), we substitute it back into (1) which gives\begin{align*} u\left ( x,t\right ) & =a_{0}\left ( t\right ) +\sum _{n=1}^{\infty }a_{n}\left ( t\right ) \Phi _{n}\left ( x\right ) \\ u\left ( x,t\right ) & =\frac{ac^{2}}{3}t+\sum _{n=1}^{\infty }\frac{\left ( -1\right ) ^{n}}{n^{4}}\frac{4ac^{4}}{k\pi ^{4}}\left ( 1-e^{-k\left ( \frac{n\pi }{c}\right ) ^{2}t}\right ) \cos \left ( \frac{n\pi }{c}x\right ) \\ & =\frac{ac^{2}}{3}t+\frac{4ac^{4}}{k\pi ^{4}}\sum _{n=1}^{\infty }\frac{\left ( -1\right ) ^{n}}{n^{4}}\left ( 1-e^{-k\left ( \frac{n\pi }{c}\right ) ^{2}t}\right ) \cos \left ( \frac{n\pi }{c}x\right ) \\ & =ac^{2}\left \{ \frac{t}{3}+\frac{4c^{2}}{k\pi ^{4}}\sum _{n=1}^{\infty }\frac{\left ( -1\right ) ^{n}}{n^{4}}\left ( 1-e^{-k\left ( \frac{n\pi }{c}\right ) ^{2}t}\right ) \cos \left ( \frac{n\pi }{c}x\right ) \right \} \end{align*}

Which is the result required to show.

2.6.7 Section 43, Problem 1

pict
Figure 2.71:Problem statement

Solution

pict
Figure 2.72:PDE and boundary conditions

Let \(u\left ( x,y\right ) =X\left ( x\right ) Y\left ( y\right ) \). The PDE becomes

\begin{align*} X^{\prime \prime }Y+Y^{\prime \prime }X & =0\\ \frac{X^{\prime \prime }}{X} & =-\frac{Y^{\prime \prime }}{Y}=-\lambda \end{align*}

Hence the eigenvalue problem is

\begin{align} X^{\prime \prime }+\lambda X & =0\tag{1}\\ X^{\prime }\left ( 0\right ) & =0\nonumber \\ X^{\prime }\left ( \pi \right ) & =0\nonumber \end{align}

And the ODE for \(Y\left ( y\right ) \) is\[ Y^{\prime \prime }-\lambda Y=0 \] We start by solving (1) to find the eigenvalues and eigenfunctions.

Case \(\lambda <0\) The solution is\begin{align*} X & =A\cosh \left ( \sqrt{-\lambda }x\right ) +B\sinh \left ( \sqrt{-\lambda }x\right ) \\ X^{\prime } & =A\sqrt{-\lambda }\sinh \left ( \sqrt{-\lambda }x\right ) +B\sqrt{-\lambda }\cosh \left ( \sqrt{-\lambda }x\right ) \end{align*}

At \(x=0\) the above becomes\[ 0=B\sqrt{-\lambda }\] Hence \(B=0\) and the solution becomes\begin{align*} X & =A\cosh \left ( \sqrt{-\lambda }x\right ) \\ X^{\prime } & =A\sqrt{-\lambda }\sinh \left ( \sqrt{-\lambda }x\right ) \end{align*}

At \(x=\pi \) the above gives\[ 0=A\sqrt{-\lambda }\sinh \left ( \sqrt{-\lambda }\pi \right ) \] For nontrivial solution \(\sinh \left ( \sqrt{-\lambda }\pi \right ) =0\) but this is not possible since \(\sinh \) is zero only when its argument is zero and this is not the case here. Hence \(\lambda <0\) is not eigenvalue.

Case \(\lambda =0\) The solution is\begin{align*} X & =Ax+B\\ X^{\prime } & =A \end{align*}

At \(x=0\) the above becomes\[ 0=A \] Hence the solution becomes\begin{align*} X & =B\\ X^{\prime } & =0 \end{align*}

At \(x=\pi \) the above gives\[ 0=0 \] Therefore \(\lambda =0\) is eigenvalue with \(X_{0}\left ( x\right ) =1\).

Case \(\lambda >0\) The solution is\begin{align*} X & =A\cos \left ( \sqrt{\lambda }x\right ) +B\sin \left ( \sqrt{\lambda }x\right ) \\ X^{\prime } & =-A\sqrt{\lambda }\sin \left ( \sqrt{\lambda }x\right ) +B\sqrt{\lambda }\cos \left ( \sqrt{\lambda }x\right ) \end{align*}

At \(x=0\) the above becomes\[ 0=B\sqrt{\lambda }\] Hence \(B=0\) and the solution becomes\begin{align*} X & =A\cos \left ( \sqrt{\lambda }x\right ) \\ X^{\prime } & =-A\sqrt{\lambda }\sin \left ( \sqrt{\lambda }x\right ) \end{align*}

At \(x=\pi \) the above gives\[ 0=-A\sqrt{\lambda }\sin \left ( \sqrt{\lambda }\pi \right ) \] For nontrivial solution \begin{align*} \sin \left ( \sqrt{\lambda }\pi \right ) & =0\\ \sqrt{\lambda }\pi & =n\pi \qquad n=1,2,3,\cdots \\ \lambda _{n} & =n^{2} \end{align*}

And the corresponding eigenfunctions \(X_{n}\left ( x\right ) =\cos \left ( nx\right ) \). Therefore in summary we have

eigenvalue eigenfunction
\(\lambda _{0}=0\) \(1\)
\(\lambda _{n}=n^{2}\qquad n=1,2,3,\cdots \) \(\cos \left ( nx\right ) \)

Hence the \(Y\left ( y\right ) \) ode becomes\begin{align*} Y^{\prime \prime }-\lambda _{n}Y & =0\\ Y^{\prime \prime }-n^{2}Y & =0 \end{align*}

The solution to the above is, when \(n=0\)\[ Y_{0}=A_{0}y+B_{0}\] When \(y=0\) the above gives \(0=B_{0}\). Hence \(Y_{0}=A_{0}y\).

When \(n>0\)\[ Y_{n}\left ( y\right ) =B_{n}\cosh \left ( ny\right ) +A_{n}\sinh \left ( ny\right ) \] When \(y=0\) the above gives \(0=B_{n}\), Hence \[ Y_{n}\left ( y\right ) =A_{n}\sinh \left ( ny\right ) \] Hence the fundamental solution is\[ u\left ( x,y\right ) =X_{n}Y_{n}\] And the general solution is the superposition of these solutions\[ u\left ( x,y\right ) =A_{0}X_{0}Y_{0}+\sum _{n=1}^{\infty }A_{n}Y_{n}X_{n}\] Therefore\begin{equation} u\left ( x,y\right ) =A_{0}y+\sum _{n=1}^{\infty }A_{n}\sinh \left ( ny\right ) \cos \left ( nx\right ) \tag{A} \end{equation} What is left is to determine \(A_{0}\) and \(A_{n}\). At \(y=\pi \) the above gives\[ f\left ( x\right ) =A_{0}\pi +\sum _{n=1}^{\infty }A_{n}\sinh \left ( n\pi \right ) \cos \left ( nx\right ) \] Multiplying both sides by \(\cos \left ( mx\right ) \) and integrating gives\begin{equation} \int _{0}^{\pi }f\left ( x\right ) \cos \left ( mx\right ) dx=\int _{0}^{\pi }A_{0}\pi \cos \left ( mx\right ) dx+\int _{0}^{\pi }\sum _{n=1}^{\infty }A_{n}\sinh \left ( n\pi \right ) \cos \left ( nx\right ) \cos \left ( mx\right ) dx\tag{1} \end{equation} For \(m=0\), (1) becomes\begin{align} \int _{0}^{\pi }f\left ( x\right ) dx & =\int _{0}^{\pi }A_{0}\pi dx\nonumber \\ \int _{0}^{\pi }f\left ( x\right ) dx & =A_{0}\pi ^{2}\nonumber \\ A_{0} & =\frac{1}{\pi ^{2}}\int _{0}^{\pi }f\left ( x\right ) dx\tag{2} \end{align}

For \(m>0\), (1) becomes\begin{align*} \int _{0}^{\pi }f\left ( x\right ) \cos \left ( mx\right ) dx & =\int _{0}^{\pi }\sum _{n=1}^{\infty }A_{n}\sinh \left ( n\pi \right ) \cos \left ( nx\right ) \cos \left ( mx\right ) dx\\ \int _{0}^{\pi }f\left ( x\right ) \cos \left ( mx\right ) dx & =A_{m}\sinh \left ( m\pi \right ) \int _{0}^{\pi }\cos ^{2}\left ( nx\right ) dx\\ & =A_{m}\sinh \left ( m\pi \right ) \frac{\pi }{2} \end{align*}

Hence\begin{equation} A_{n}=\frac{2}{\pi \sinh \left ( n\pi \right ) }\int _{0}^{\pi }f\left ( x\right ) \cos \left ( nx\right ) dx\tag{3} \end{equation} When \(f\left ( x\right ) =u_{0}\) a constant, then (2) becomes\begin{align*} A_{0} & =\frac{1}{\pi ^{2}}\int _{0}^{\pi }u_{0}dx\\ & =\frac{u_{0}}{\pi } \end{align*}

And (3) becomes\begin{align*} A_{n} & =\frac{2}{\pi \sinh \left ( n\pi \right ) }\int _{0}^{\pi }u_{0}\cos \left ( nx\right ) dx\\ & =\frac{2u_{0}}{\pi \sinh \left ( n\pi \right ) }\left [ \frac{\sin \left ( nx\right ) }{n}\right ] _{0}^{\pi }\\ & =0 \end{align*}

Hence the solution (A) becomes\[ \fbox{$u\left ( x,y\right ) =u_0\frac{y}{\pi }$}\] This shows the final solution changes linearly in \(y\). When \(y=0\) then \(u\left ( x,0\right ) =0\) and when \(y=\pi \), then \(u\left ( x,\pi \right ) =u_{0}\).

2.6.8 Section 44, Problem 2

pict
Figure 2.73:Problem statement

Solution

The PDE \(\nabla ^{2}u\left ( \rho ,\phi \right ) =0\) in polar coordinates is \[ u_{\rho \rho }+\frac{1}{\rho }u_{\rho }+\frac{1}{\rho ^{2}}u_{\phi \phi }=0 \] For \(0<\rho <a\) and \(0<\phi <\alpha \). With boundary conditions\begin{align*} u\left ( \rho ,0\right ) & =0\\ u\left ( \rho ,\alpha \right ) & =0\\ u\left ( a,\phi \right ) & =f\left ( \phi \right ) \end{align*}

And since \(u\) is bounded, then we have an extra condition \(u\left ( 0,\phi \right ) <\infty \).

Let \(u\left ( \rho ,\phi \right ) =R\left ( \rho \right ) \Phi \left ( \phi \right ) \). Substituting into the above PDE gives\begin{align*} R^{\prime \prime }\Phi +\frac{1}{\rho }R^{\prime }\Phi +\frac{1}{\rho ^{2}}\Phi ^{\prime \prime }R & =0\\ \frac{R^{\prime \prime }}{R}+\frac{1}{\rho }\frac{R^{\prime }}{R}+\frac{1}{\rho ^{2}}\frac{\Phi ^{\prime \prime }}{\Phi } & =0\\ \frac{\Phi ^{\prime \prime }}{\Phi } & =-\left ( \rho ^{2}\frac{R^{\prime \prime }}{R}+\rho \frac{R^{\prime }}{R}\right ) =-\lambda \end{align*}

Where \(\lambda \) is the separation constant. The above gives the boundary values problem to solve for \(\lambda \)\begin{align} \Phi ^{\prime \prime }+\lambda \Phi & =0\tag{1}\\ \Phi \left ( 0\right ) & =0\nonumber \\ \Phi \left ( \alpha \right ) & =0\nonumber \end{align}

And\begin{align} \rho ^{2}\frac{R^{\prime \prime }}{R}+\rho \frac{R^{\prime }}{R} & =\lambda \nonumber \\ \rho ^{2}R^{\prime \prime }+\rho R^{\prime }-\lambda R & =0\tag{2} \end{align}

We start with (1) to find \(\lambda \) then use the result to solve (2). The ODE (1) we solved before, it has the eigenvalues \[ \lambda _{n}=\left ( \frac{n\pi }{\alpha }\right ) ^{2}\qquad n=1,2,3,\cdots \] And corresponding eigenfunctions\begin{equation} \Phi _{n}\left ( \phi \right ) =\sin \left ( \frac{n\pi }{\alpha }\phi \right ) \tag{3} \end{equation} Now (2) can be solved. This is a Euler ODE. Using \(R\left ( \rho \right ) =\rho ^{m}\) and substituting into (2) gives\begin{align*} \rho ^{2}m\left ( m-1\right ) \rho ^{m-2}+\rho m\rho ^{m-1}-\left ( \frac{n\pi }{\alpha }\right ) ^{2}\rho ^{m} & =0\\ m\left ( m-1\right ) \rho ^{m}+m\rho ^{m}-\left ( \frac{n\pi }{\alpha }\right ) ^{2}\rho ^{m} & =0\\ m\left ( m-1\right ) +m-\left ( \frac{n\pi }{\alpha }\right ) ^{2} & =0\\ m^{2} & =\left ( \frac{n\pi }{\alpha }\right ) ^{2} \end{align*}

Hence \[ m=\pm \frac{n\pi }{\alpha }\] Therefore the solution to (2) is\[ R_{n}\left ( \rho \right ) =A_{n}\rho ^{\frac{n\pi }{\alpha }}+B_{n}\rho ^{\frac{-n\pi }{\alpha }}\] We immediately reject the solution \(\rho ^{\frac{-n\pi }{\alpha }}\) since this blows up at origin where \(\rho \rightarrow 0\). Hence the above becomes\begin{equation} R_{n}\left ( \rho \right ) =A_{n}\rho ^{\frac{n\pi }{\alpha }}\tag{4} \end{equation} Now that we found \(\Phi _{n}\left ( \phi \right ) \) and \(R_{n}\left ( \rho \right ) \), then we use superposition to obtain the general solution\begin{align} u\left ( \rho ,\phi \right ) & =\sum _{n=1}^{\infty }R_{n}\left ( \rho \right ) \Phi _{n}\left ( \phi \right ) \nonumber \\ & =\sum _{n=1}^{\infty }A_{n}\rho ^{\frac{n\pi }{\alpha }}\sin \left ( \frac{n\pi }{\alpha }\phi \right ) \tag{5} \end{align}

At \(\rho =a\), \(u\left ( a,\phi \right ) =f\left ( \phi \right ) \), hence the above becomes\[ f\left ( \phi \right ) =\sum _{n=1}^{\infty }A_{n}a^{\frac{n\pi }{\alpha }}\sin \left ( \frac{n\pi }{\alpha }\phi \right ) \] By orthogonality we obtain\begin{align*} \int _{0}^{\alpha }f\left ( \phi \right ) \sin \left ( \frac{m\pi }{\alpha }\phi \right ) d\phi & =\int _{0}^{\alpha }\sum _{n=1}^{\infty }A_{n}a^{\frac{n\pi }{\alpha }}\sin \left ( \frac{n\pi }{\alpha }\phi \right ) \sin \left ( \frac{m\pi }{\alpha }\phi \right ) d\phi \\ & =A_{m}a^{\frac{m\pi }{\alpha }}\int _{0}^{\alpha }\sin ^{2}\left ( \frac{m\pi }{\alpha }\phi \right ) d\phi \\ & =A_{m}a^{\frac{m\pi }{\alpha }}\frac{\alpha }{2} \end{align*}

Solving for \(A_{n}\) from the above gives\[ A_{n}=\frac{2}{\alpha }a^{\frac{-n\pi }{\alpha }}\int _{0}^{\alpha }f\left ( \phi \right ) \sin \left ( \frac{n\pi }{\alpha }\phi \right ) d\phi \] Substituting the above in (5) gives the final solution\begin{align*} u\left ( \rho ,\phi \right ) & =\sum _{n=1}^{\infty }\left ( \frac{2}{\alpha }a^{\frac{-n\pi }{\alpha }}\int _{0}^{\alpha }f\left ( \psi \right ) \sin \left ( \frac{n\pi }{\alpha }\psi \right ) d\psi \right ) \rho ^{\frac{n\pi }{\alpha }}\sin \left ( \frac{n\pi }{\alpha }\phi \right ) \\ & =\frac{2}{\alpha }\sum _{n=1}^{\infty }\left ( \frac{\rho }{a}\right ) ^{\frac{n\pi }{\alpha }}\sin \left ( \frac{n\pi }{\alpha }\phi \right ) \left ( \int _{0}^{\alpha }f\left ( \psi \right ) \sin \left ( \frac{n\pi }{\alpha }\psi \right ) d\psi \right ) \end{align*}

2.6.9 Section 49, Problem 2

pict

pict
Figure 2.74:Problem statement

Solution

\[ u_{t}=ku_{xx}\] With \(-\pi <x<\pi ,t>0\) and periodic boundary conditions\begin{align*} u\left ( -\pi ,t\right ) & =u\left ( \pi ,t\right ) \\ u_{x}\left ( -\pi ,t\right ) & =u_{x}\left ( \pi ,t\right ) \end{align*}

And initial conditions\[ u\left ( x,0\right ) =f\left ( x\right ) \] Normal process of separation of variables leads to eigenvalue problem\begin{align} X^{\prime \prime }+\lambda X & =0\tag{1}\\ X\left ( -\pi \right ) & =X\left ( \pi \right ) \nonumber \\ X^{\prime }\left ( -\pi \right ) & =X^{\prime }\left ( \pi \right ) \nonumber \end{align}

And the time ODE\begin{equation} T^{\prime }+k\lambda T=0\tag{2} \end{equation} We start by solving (1) to find the eigenvalues and eigenfunctions.

Case \(\lambda <0\)

Solution is \begin{align*} X\left ( x\right ) & =A\cosh \left ( \sqrt{-\lambda }x\right ) +B\sinh \left ( \sqrt{-\lambda }x\right ) \\ X^{\prime }\left ( x\right ) & =A\sqrt{-\lambda }\sinh \left ( \sqrt{-\lambda }x\right ) +B\sqrt{-\lambda }\cosh \left ( \sqrt{-\lambda }x\right ) \end{align*}

The boundary conditions \(X\left ( -\pi \right ) =X\left ( \pi \right ) \) results in (using the fact that \(\cosh \) is even and \(\sinh \) is odd)\begin{align} A\cosh \left ( \sqrt{-\lambda }\pi \right ) +B\sinh \left ( \sqrt{-\lambda }\pi \right ) & =A\cosh \left ( \sqrt{-\lambda }\pi \right ) -B\sinh \left ( \sqrt{-\lambda }\pi \right ) \nonumber \\ B\sinh \left ( \sqrt{-\lambda }\pi \right ) & =-B\sinh \left ( \sqrt{-\lambda }\pi \right ) \nonumber \\ B\sinh \left ( \sqrt{-\lambda }\pi \right ) & =0\tag{3} \end{align}

The boundary conditions \(X^{\prime }\left ( -\pi \right ) =X^{\prime }\left ( \pi \right ) \) results in (using the fact that \(\cosh \) is even and \(\sinh \) is odd)\begin{align} A\sqrt{-\lambda }\sinh \left ( \sqrt{-\lambda }\pi \right ) +B\sqrt{-\lambda }\cosh \left ( \sqrt{-\lambda }\pi \right ) & =-A\sqrt{-\lambda }\sinh \left ( \sqrt{-\lambda }\pi \right ) +B\sqrt{-\lambda }\cosh \left ( \sqrt{-\lambda }\pi \right ) \nonumber \\ A\sqrt{-\lambda }\sinh \left ( \sqrt{-\lambda }\pi \right ) & =-A\sqrt{-\lambda }\sinh \left ( \sqrt{-\lambda }\pi \right ) \nonumber \\ A\sinh \left ( \sqrt{-\lambda }\pi \right ) & =0\tag{4} \end{align}

So we obtain (3,4) equations, here they are again\begin{align*} B\sinh \left ( \sqrt{-\lambda }\pi \right ) & =0\\ A\sinh \left ( \sqrt{-\lambda }\pi \right ) & =0 \end{align*}

There are two possibility, either \(\sinh \left ( \sqrt{-\lambda }\pi \right ) =0\) or \(\sinh \left ( \sqrt{-\lambda }\pi \right ) \neq 0\). If \(\sinh \left ( \sqrt{-\lambda }\pi \right ) \neq 0\) then this leads to trivial solution, as it implies that both \(A=0\) and \(B=0\). On the other hand, if \(\sinh \left ( \sqrt{-\lambda }\pi \right ) =0\) then this implies that \(\sqrt{-\lambda }\pi =0\) since \(\sinh \) is only zero when its argument is zero which is not the case here. This implies that \(\lambda <0\) is not possible.

Case \(\lambda =0\)

The solution now becomes \(X\left ( x\right ) =Ax+B\). Satisfying the boundary conditions \(X\left ( -\pi \right ) =X\left ( \pi \right ) \) gives\begin{align*} A\pi +B & =-A\pi +B\\ 2A\pi & =0\\ A & =0 \end{align*}

Hence the solution becomes\begin{align*} X\left ( x\right ) & =B\\ X^{\prime } & =0 \end{align*}

Satisfying the boundary conditions \(X^{\prime }\left ( -\pi \right ) =X^{\prime }\left ( \pi \right ) \) gives \(0=0\). Hence \(\lambda =0\) is possible eigenvalue, with corresponding eigenfunction as constant, say \(1\).

Case \(\lambda >0\)

Solution is \begin{align*} X\left ( x\right ) & =A\cos \left ( \sqrt{\lambda }x\right ) +B\sin \left ( \sqrt{\lambda }x\right ) \\ X^{\prime }\left ( x\right ) & =-A\sqrt{\lambda }\sin \left ( \sqrt{\lambda }x\right ) +B\sqrt{\lambda }\cos \left ( \sqrt{\lambda }x\right ) \end{align*}

The boundary conditions \(X\left ( -\pi \right ) =X\left ( \pi \right ) \) results in (using the fact that \(\cos \) is even and \(\sin \) is odd)\begin{align} A\cos \left ( \sqrt{\lambda }\pi \right ) +B\sin \left ( \sqrt{\lambda }\pi \right ) & =A\cos \left ( \sqrt{\lambda }\pi \right ) -B\sin \left ( \sqrt{\lambda }\pi \right ) \nonumber \\ B\sin \left ( \sqrt{\lambda }\pi \right ) & =-B\sin \left ( \sqrt{\lambda }\pi \right ) \nonumber \\ B\sin \left ( \sqrt{\lambda }\pi \right ) & =0\tag{5} \end{align}

The boundary conditions \(X^{\prime }\left ( -\pi \right ) =X^{\prime }\left ( \pi \right ) \) results in (using the fact that \(\cosh \) is even and \(\sinh \) is odd)\begin{align} -A\sqrt{\lambda }\sin \left ( \sqrt{\lambda }\pi \right ) +B\sqrt{\lambda }\cos \left ( \sqrt{\lambda }\pi \right ) & =A\sqrt{\lambda }\sin \left ( \sqrt{\lambda }\pi \right ) +B\sqrt{\lambda }\cos \left ( \sqrt{\lambda }\pi \right ) \nonumber \\ -A\sqrt{\lambda }\sin \left ( \sqrt{\lambda }\pi \right ) & =A\sqrt{\lambda }\sin \left ( \sqrt{\lambda }\pi \right ) \nonumber \\ A\sin \left ( \sqrt{\lambda }\pi \right ) & =0\tag{6} \end{align}

So we obtain (5,6) equations, here they are again\begin{align*} B\sin \left ( \sqrt{\lambda }\pi \right ) & =0\\ A\sin \left ( \sqrt{\lambda }\pi \right ) & =0 \end{align*}

There are two possibility, either \(\sin \left ( \sqrt{\lambda }\pi \right ) =0\) or \(\sin \left ( \sqrt{\lambda }\pi \right ) \neq 0\). If \(\sin \left ( \sqrt{\lambda }\pi \right ) \neq 0\) then this leads to trivial solution, as it implies that both \(A=0\) and \(B=0\). If \(\sin \left ( \sqrt{\lambda }\pi \right ) =0\) then this implies that \(\sqrt{\lambda }\pi =n\pi \) where \(n=1,2,3,\cdots \). Hence \(\lambda >0\) is possible with eigenvalues and corresponding eigenfunctions given by \begin{align*} \lambda _{n} & =n^{2}\qquad n=1,2,3,\cdots \\ X_{n}\left ( x\right ) & =A_{n}\cos \left ( nx\right ) +B_{n}\sin \left ( nx\right ) \end{align*}

Now that we solved the eigenvalue problem (1), we use the eigenvalues found to solve the time ODE (2)\[ T^{\prime }+k\lambda _{n}T=0 \] When \(\lambda =0\), this becomes \(T^{\prime }=0\) or \(T_{0}\left ( t\right ) \) is constant. When \(\lambda >0\) the solution is\begin{align*} T_{n}\left ( t\right ) & =e^{-k\lambda _{n}t}\\ & =e^{-kn^{2}t} \end{align*}

Hence the fundamental solution is\[ u_{n}\left ( x,t\right ) =X_{n}\left ( x\right ) T_{n}\left ( t\right ) \] And by superposition, the general solution is\[ u\left ( x,t\right ) =A_{0}X_{0}\left ( x\right ) T_{0}\left ( t\right ) +\sum _{n=1}^{\infty }\left ( A_{n}\cos \left ( nx\right ) +B_{n}\sin \left ( nx\right ) \right ) e^{-kn^{2}t}\] But \(X_{0}\left ( x\right ) =1\) and \(T_{0}\left ( t\right ) \) is constant. Hence the above simplifies to\[ u\left ( x,t\right ) =A_{0}+\sum _{n=1}^{\infty }\left ( A_{n}\cos \left ( nx\right ) +B_{n}\sin \left ( nx\right ) \right ) e^{-kn^{2}t}\] What is left is to find \(A_{0},A_{n},B_{n}\). At \(t=0\) the above gives\begin{equation} f\left ( x\right ) =A_{0}+\sum _{n=1}^{\infty }A_{n}\cos \left ( nx\right ) +B_{n}\sin \left ( nx\right ) \tag{7} \end{equation} For \(n=0\), by orthogonality we obtain\begin{align*} \int _{-\pi }^{\pi }f\left ( x\right ) dx & =\int _{-\pi }^{\pi }A_{0}dx\\ \int _{-\pi }^{\pi }f\left ( x\right ) dx & =A_{0}\left ( 2\pi \right ) \\ A_{0} & =\frac{1}{2\pi }\int _{-\pi }^{\pi }f\left ( x\right ) dx \end{align*}

For \(n>0\). We start by multiplying both sides of (7) by \(\cos \left ( mx\right ) \) and integrating both sides. This gives\begin{align*} \int _{-\pi }^{\pi }f\left ( x\right ) \cos \left ( mx\right ) dx & =\int _{-\pi }^{\pi }\left ( \sum _{n=1}^{\infty }A_{n}\cos \left ( nx\right ) \cos \left ( mx\right ) +B_{n}\sin \left ( nx\right ) \cos \left ( mx\right ) \right ) dx\\ & =\sum _{n=1}^{\infty }A_{n}\int _{-\pi }^{\pi }\cos \left ( nx\right ) \cos \left ( mx\right ) dx+\sum _{n=1}^{\infty }B_{n}\int _{-\pi }^{\pi }\sin \left ( nx\right ) \cos \left ( mx\right ) dx \end{align*}

But \(\int _{-\pi }^{\pi }\sin \left ( nx\right ) \cos \left ( mx\right ) dx=0\) for all \(n,m\). And \(\int _{-\pi }^{\pi }\cos \left ( nx\right ) \cos \left ( mx\right ) dx=\int _{-\pi }^{\pi }\cos ^{2}\left ( mx\right ) dx\) and zero for all other \(n\neq m\). Hence the above simplifies to\begin{align*} \int _{-\pi }^{\pi }f\left ( x\right ) \cos \left ( mx\right ) dx & =A_{m}\int _{-\pi }^{\pi }\cos ^{2}\left ( mx\right ) dx\\ & =A_{m}\pi \end{align*}

Therefore\[ A_{n}=\frac{1}{\pi }\int _{-\pi }^{\pi }f\left ( x\right ) \cos \left ( nx\right ) dx \] To find \(B_{n}\) we do the same, but now we multiply both sides of (7) by \(\sin \left ( mx\right ) \) and this leads to \begin{align*} \int _{-\pi }^{\pi }f\left ( x\right ) \sin \left ( mx\right ) dx & =\int _{-\pi }^{\pi }\left ( \sum _{n=1}^{\infty }A_{n}\cos \left ( nx\right ) \sin \left ( mx\right ) +B_{n}\sin \left ( nx\right ) \sin \left ( mx\right ) \right ) dx\\ & =\sum _{n=1}^{\infty }A_{n}\int _{-\pi }^{\pi }\cos \left ( nx\right ) \sin \left ( mx\right ) dx+\sum _{n=1}^{\infty }B_{n}\int _{-\pi }^{\pi }\sin \left ( nx\right ) \sin \left ( mx\right ) dx \end{align*}

But \(\int _{-\pi }^{\pi }\cos \left ( nx\right ) \sin \left ( mx\right ) dx=0\) for all \(n,m\). And \(\int _{-\pi }^{\pi }\sin \left ( nx\right ) \sin \left ( mx\right ) dx=\int _{-\pi }^{\pi }\sin ^{2}\left ( mx\right ) dx\) and zero for all other \(n\neq m\). Hence the above simplifies to\begin{align*} \int _{-\pi }^{\pi }f\left ( x\right ) \sin \left ( mx\right ) dx & =B_{m}\int _{-\pi }^{\pi }\sin ^{2}\left ( mx\right ) dx\\ & =B_{m}\pi \end{align*}

Therefore\[ B_{n}=\frac{1}{\pi }\int _{-\pi }^{\pi }f\left ( x\right ) \sin \left ( nx\right ) dx \] This completes the solution. The final solution is \begin{align*} u\left ( x,t\right ) & =A_{0}+\sum _{n=1}^{\infty }\left ( A_{n}\cos \left ( nx\right ) +B_{n}\sin \left ( nx\right ) \right ) e^{-kn^{2}t}\\ & =\frac{1}{2\pi }\int _{-\pi }^{\pi }f\left ( x\right ) dx+\sum _{n=1}^{\infty }e^{-kn^{2}t}\left [ \left ( \frac{1}{\pi }\int _{-\pi }^{\pi }f\left ( x\right ) \cos \left ( nx\right ) dx\right ) \cos \left ( nx\right ) +\left ( \frac{1}{\pi }\int _{-\pi }^{\pi }f\left ( x\right ) \sin \left ( nx\right ) dx\right ) \sin \left ( nx\right ) \right ] \end{align*}